12
$\begingroup$

Let $X$ and $Y$ be independent random variates with the same probability distribution, $P(x)$. Assuming that the product $Z=XY$ is a random variate with normal distribution, say $$f_Z(x) = \frac{1}{\sqrt{2\pi}} e^{-\frac{1}{2} x^2}$$ what is the form of $P(x)$? Does it exist in closed form?

EDIT: I just realized that a product distribution is necessarily divergent at zero, which means that the normal distribution cannot be a product distribution (at least when $P(x)$ is interpreted as a normal function; maybe there is a generalized function that solves the problem?). Do you agree?

$\endgroup$
3
  • $\begingroup$ I guess we would just need $$ \frac{1}{\sqrt{2\pi}}e^{-z^2/2}=\int_{-\infty}^\infty f_X(x)\,f_X(z/x)\,\frac1{|x|}\,dx $$ $\endgroup$ Mar 11, 2015 at 19:51
  • $\begingroup$ The existence of the square root follows from the infinite divisibility of the log-normal distribution, does it not? tandfonline.com/doi/abs/10.1080/03461238.1977.10405635 $\endgroup$ Mar 13, 2015 at 1:00
  • $\begingroup$ Brendan: I think log-normal distribution is in the opposite direction -- it is the distribution of $e^{tZ}$ for a nonzero real $t$, whereas here of relevance would be the infinite divisibility of $\ln|Z|$. $\endgroup$ Mar 10, 2016 at 2:22

1 Answer 1

10
$\begingroup$

First: No helvio, the density of the product of two iid random variables (r.v.'s) is not necessarily divergent at 0. E.g., take the product of two iid r.v.'s each having a Gamma distribution with shape parameter 2.

Second: Let $U:=\ln|Z|$ and $W:=\ln|X|$. Then the characteristic function (c.f.) $f$ of $U$ is given by the formula $f(t)=2^{i t/2} \Gamma((1+i t)/2)/\sqrt{\pi}$ for real $t$. So, the c.f. of $W$ (say $g$) would be a square root of $f$, and then the density (say $q$) of $W$ would be given by the formula $q(x)=\frac1{2\pi}\,\int_{-\infty }^{\infty } g(t) e^{-i t x} \, dt$ for real $x$. I have tried to evaluate the latter integral numerically, and that seems to suggest that $q(3)<0$; however, I have not done a rigorous estimate of the approximation error.

On the other hand, numerical experiments using Bochner's theorem on the positive definite functions suggest that $g$ is a c.f.! If so, then the corresponding r.v. $W$ exists, and then it suffices to let $X$ have the distribution of $\varepsilon e^W$, where $\varepsilon$ is a Rademacher r.v. independent of $W$.

Addendum: The calculation of $q(3)$ mentioned previously was likely mistaken, as I forgot to make sure that $g(t)$ be taken to be the value of the square root of $z:=f(t)$ on the Riemann surface for $\sqrt z$, so as for $g$ to be a continuous function -- as it must be.

Another approach to the problem: Note that $\mu_j:=E|Z|^j=2^{j/2} \Gamma \left(\frac{j+1}{2}\right)/\sqrt{\pi }$ and then one would have $\nu_j:=E|X|^j=\sqrt{\mu_j}$ for $j>0$. To prove that such a desired r.v. $X$ exists, it is enough to show that for all nonnegative integers $n$ the determinants, say $d_n$ and $e_n$, of the matrices $(\nu_{i+j})_{i,j=0}^n$ and $(\nu_{i+j+1})_{i,j=0}^n$ are nonnegative (see e.g. Ch. V in Kreǐn, M. G. and Nudel'man, A. A., The Markov moment problem and extremal problems). These determinants are indeed positive for $n\le9$. One can hopefully find a recursion for $d_n$ and $e_n$ to show that they are indeed nonnegative for all $n$. Note here that $\mu_j$ is an integer or an integral multiple of $\sqrt{2/\pi}$.

$\endgroup$
3
  • 1
    $\begingroup$ Thank you very much @iosif-pinelis for your detailed answer, I learned a great deal with it. So if I understood correctly the probability density $P(x)$ is given by the Fourier transform of the square root of the characteristic function of the normal probability density, $$P(x) = \int_{-\infty}^{+\infty} \left( \frac{2^{it/2}}{\sqrt\pi} \Gamma\Big(\frac{1+it}{2}\Big) \right)^\frac{1}{2} e^{-itx} dt$$ right? I can't expect a closed form solution, and even if it existed it's probably useless in practice. But the whole argument is sound and solid, so I'll accept it. $\endgroup$
    – FreeQuark
    Mar 13, 2015 at 13:46
  • 2
    $\begingroup$ helvio: $\frac{P(x)}{2\pi}$ would be the density of $\ln|X|$ -- provided that $P(x)$ is nonnegative for all real $x$. I think this nonnegativity is likely the case, but don't know how to prove that. The other approach that I suggested later, with the $\mu_j$'s and $\nu_j$'s, may be more promising. $\endgroup$ Mar 13, 2015 at 16:58
  • 4
    $\begingroup$ A complete answer to this question (and actually a more general one) is now available at arxiv.org/abs/1803.09838 $\endgroup$ Apr 2, 2018 at 0:08

Your Answer

By clicking “Post Your Answer”, you agree to our terms of service and acknowledge you have read our privacy policy.

Not the answer you're looking for? Browse other questions tagged or ask your own question.